Está en la página 1de 15

Microeconomic Theory - Fall 2011

Collegio Carlo Alberto


Prof. Ignacio Monz on
PS #4: EXPENDITURE MINIMIZATION PROBLEM AND MORE DEMAND THEORY
DUE DATE: 11 NOV 2011
ANSWER KEY
1. MWG 3.D.6
B
Consider the three-good setting in which the consumer has utility function
u(x) = (x
1
b
1
)

(x
2
b
2
)

(x
3
b
3
)

.
(a) Why can you assume that + + = 1 without loss of generality? Do so for the rest
of the problem.
Answer: We assume that , , > 0, and so + + > 0. Then, we can dene a new
utility function that will also represent preferences by v(x) = [u(x)]
1
++
, which will
make the exponents add up to 1. From now on, we do so, and we take natural logs, so
we write the utility function as follows,
u(x) = ln (x
1
b
1
) + ln (x
2
b
2
) + (1 ) ln (x
3
b
3
)
(b) Write down the rst order conditions for the UMP, and derive the consumers Wal-
rasian demand and indirect utility functions. This system of demand is known as linear
expenditure system and is due to Stone (1954)
Answer: The utility maximization problem can be solved using a Lagrangian:
L(x) = ln (x
1
b
1
) + ln (x
2
b
2
) + (1 ) ln (x
3
b
3
) + (w p
1
x
1
p
2
x
2
p
3
x
3
)
Consequently, the rst order conditions are given by:
L
x
1
=

x
1
b
1
p
1
= 0
L
x
2
=

x
2
b
2
p
2
= 0
L
x
3
=
1
x
3
b
3
p
3
= 0
L

= w p
1
x
1
p
2
x
2
p
3
x
3
= 0
As a result,

p
1
(x
1
b
1
)
=

p
2
(x
2
b
2
)
=
1
p
3
(x
3
b
3
)
Or, to make our life easier,
p
1
(x
1
b
1
)

=
p
2
(x
2
b
2
)

=
p
3
(x
3
b
3
)
1
In this way, it makes sense to express both x
2
and x
3
as functions of x
1
:
p
2
x
2
= p
2
b
2
+

p
1
x
1

p
1
b
1
p
3
x
3
= p
3
b
3
+
1

p
1
x
1

p
1
b
1
Now, we can replace in the fourth FOC,
p
1
x
1
+ p
2
x
2
+ p
3
x
3
= w
p
1
x
1
+ p
2
b
2
+

p
1
x
1

p
1
b
1
+ p
3
b
3
+
1

p
1
x
1

p
1
b
1
= w
p
2
b
2
+ p
3
b
3
+
+ + 1

p
1
x
1

1 +

p
1
b
1
= w
p
2
b
2
+ p
3
b
3
+
1

p
1
x
1

p
1
b
1
= w
And then,
x
1
(p, w) =
1
p
1
[ (w p
2
b
2
p
3
b
3
) + (1 ) p
1
b
1
]
As a result,
p
2
x
2
= p
2
b
2
+

p
1
x
1

p
1
b
1
= p
2
b
2
+

[ (w p
2
b
2
p
3
b
3
) + (1 ) p
1
b
1
]

p
1
b
1
= p
2
b
2
+ (w p
2
b
2
p
3
b
3
) +

(1 1) p
1
b
1
= (1 )p
2
b
2
+ (w p
1
b
1
p
3
b
3
)
And then,
x
2
(p, w) =
1
p
2
[ (w p
1
b
1
p
3
b
3
) + (1 ) p
2
b
2
]
Finally, in a similar fashion, one can get,
x
3
(p, w) =
1
p
3
[(1 ) (w p
1
b
1
p
2
b
2
) + ( + ) p
3
b
3
]
Next, regarding the indirect utility function v(p, w), one needs only to remember that
v(p, w) = u (x(p, w)). Then,
v(p, w) =
_
1
p
1
[ (w p
2
b
2
p
3
b
3
) + (1 ) p
1
b
1
] b
1
_

_
1
p
2
[ (w p
1
b
1
p
3
b
3
) + (1 ) p
2
b
2
] b
2
_

_
1
p
3
[(1 ) (w p
1
b
1
p
2
b
2
) + ( + ) p
3
b
3
] b
3
_
1
=
_

_
w
p
1

p
2
p
1
b
2

p
3
p
1
b
3
b
1
__

_
w
p
2

p
1
p
2
b
1

p
3
p
2
b
3
b
2
__

_
(1 )
_
w
p
3

p
1
p
3
b
1

p
2
p
3
b
2
b
3
__
1
Finally,
v(p, w) =
_

(1 )
1
_ _
p

1
p

2
p
(1)
3
_
(w p
1
b
1
p
2
b
2
p
3
b
3
)
(c) Verify that these demand functions satisfy the properties listed in Propositions 3.D.2
and 3.D.3.
Answer: Next, we check that the properties from Proposition 3.D.2 hold.
i. Homogeneity of degree zero in (p, w). Ill check it only for x
1
(p, w).
x
1
(tp, tw) =
1
tp
1
[ (tw tp
2
b
2
tp
3
b
3
) + (1 ) tp
1
b
1
]
=
t
tp
1
[ (w p
2
b
2
p
3
b
3
) + (1 ) p
1
b
1
]
=
1
p
1
[ (w p
2
b
2
p
3
b
3
) + (1 ) p
1
b
1
]
= x
1
(p, w)
ii. Walras law:
w = p
1
x
1
(p, w) + p
2
x
2
(p, w) + p
3
x
3
(p, w)
= p
1
1
p
1
[ (w p
2
b
2
p
3
b
3
) + (1 ) p
1
b
1
] + p
2
1
p
2
[ (w p
1
b
1
p
3
b
3
) + (1 ) p
2
b
2
]
+ p
3
1
p
3
[(1 ) (w p
1
b
1
p
2
b
2
) + ( + ) p
3
b
3
]
= (w p
2
b
2
p
3
b
3
) + (1 ) p
1
b
1
+ (w p
1
b
1
p
3
b
3
) + (1 ) p
2
b
2
+ (1 ) (w p
1
b
1
p
2
b
2
) + ( + ) p
3
b
3
= w p
2
b
2
p
3
b
3
+ (1 ) p
1
b
1
p
1
b
1
p
3
b
3
+ (1 ) p
2
b
2
(1 ) p
1
b
1
(1 ) p
2
b
2
+ ( + ) p
3
b
3
= w
iii. Uniqueness. Since the demands we found are unique, we are done.
Finally, we check that the properties from Proposition 3.D.3 hold.
i. Homogeneity of degree zero in (p, w). Let J =
_

(1 )
1

. Then, we
can rewrite v(p, w) as follows:
v(p, w) = J
_
p

1
p

2
p
(1)
3
_
(w p
1
b
1
p
2
b
2
p
3
b
3
)
Then,
v(tp, tw) = J
_
(tp
1
)

(tp
2
)

(tp
3
)
(1)
_
(tw tp
1
b
1
tp
2
b
2
tp
3
b
3
)
= J t
1++
_
p

1
p

2
p
(1)
3
_
t (w p
1
b
1
p
2
b
2
p
3
b
3
)
= J
_
p

1
p

2
p
(1)
3
_
(w p
1
b
1
p
2
b
2
p
3
b
3
)
= v(p, w)
3
ii. Strictly increasing in w and nonincreasing in p
l
for all l. Again, recall that:
v(p, w) = J
_
p

1
p

2
p
(1)
3
_
(w p
1
b
1
p
2
b
2
p
3
b
3
)
Note:
v(p, w)
w
= J
_
p

1
p

2
p
(1)
3
_
> 0
Next,
v(p, w)
p
1
= J p

2
p
(1)
3
_
p
1
1
(w p
1
b
1
p
2
b
2
p
3
b
3
) b
1
p

1
_
< 0
The same holds for p
2
and p
3
.
iii. Quasiconvexity. Take any (p, w) and (p

, w

) such that v (p, w) v and v (p

, w

)
v. Pick a convex combination (p

, w

) = (tp + (1 t)p

, tw + (1 t)w

) for t
[0, 1]. Then:
v
_
p

, w

_
= J
tw + (1 t)w

(tp
1
+ (1 t)p

1
) b
1
(tp
2
+ (1 t)p

2
) b
2
(tp
3
+ (1 t)p

3
) b
3
_
tp
1
+ (1 t)p

1
_

(tp
2
+ (1 t)p

2
)

(tp
3
+ (1 t)p

3
)
(1)
=
tJ (w p
1
b
1
p
2
b
2
p
3
b
3
) + J(1 t) (w

1
b
1
p

2
b
2
p

3
b
3
)
_
tp
1
+ (1 t)p

1
_

(tp
2
+ (1 t)p

2
)

(tp
3
+ (1 t)p

3
)
(1)
=
tJ
wp
1
b
1
p
2
b
2
p
3
b
3
p

1
p

2
p

3
_
p

1
p

2
p

3
_
+ (1 t)J
w

1
b
1
p

2
b
2
p

3
b
3
(p

1
)

(p

2
)

(p

3
)

_
(p

1
)

(p

2
)

(p

3
)

_
_
tp
1
+ (1 t)p

1
_

(tp
2
+ (1 t)p

2
)

(tp
3
+ (1 t)p

3
)
(1)
=
tv (p, w)
_
p

1
p

2
p

3
_
+ (1 t)v (p

, w

)
_
(p

1
)

(p

2
)

(p

3
)

_
_
tp
1
+ (1 t)p

1
_

(tp
2
+ (1 t)p

2
)

(tp
3
+ (1 t)p

3
)
(1)
v
t
_
p

1
p

2
p

3
_
+ (1 t)
_
(p

1
)

(p

2
)

(p

3
)

_
_
tp
1
+ (1 t)p

1
_

(tp
2
+ (1 t)p

2
)

(tp
3
+ (1 t)p

3
)
(1)
So it sufces to show that:
t
_
p

1
p

2
p

3
_
+ (1 t)
_
(p

1
)

(p

2
)

(p

3
)

_
_
tp
1
+ (1 t)p

1
_

(tp
2
+ (1 t)p

2
)

(tp
3
+ (1 t)p

3
)
(1)
1
But that is just the condition that p

1
p

2
p

3
be concave, which it is. So v(p, w) is
quasiconvex.
iv. Continuity in p and w. Since all parts are continuous, v is continuous.
4
2. MWG 3.G.3
B
Consider the (linear expenditure system) utility function given in Exercise
3.D.6.
(a) Derive the Hicksean demand and expenditure functions. Check the properties listed in
Propositions 3.E.2 and 3.E.3.
Answer: The expenditure minimization problem can be solved using a Lagrangian:
L(x) = p
1
x
1
+ p
2
x
2
+ p
3
x
3
+ [u ln (x
1
b
1
) ln (x
2
b
2
) (1 ) ln (x
3
b
3
)]
Consequently, the rst order conditions are given by:
L
x
1
= p
1


x
1
b
1
= 0
L
x
2
= p
2


x
2
b
2
= 0
L
x
3
= p
3

1
x
3
b
3
= 0
L

= u ln (x
1
b
1
) ln (x
2
b
2
) (1 ) ln (x
3
b
3
) = 0
As a result,

p
1
(x
1
b
1
)
=

p
2
(x
2
b
2
)
=
1
p
3
(x
3
b
3
)
Or, to make our life easier,
p
1
(x
1
b
1
)

=
p
2
(x
2
b
2
)

=
p
3
(x
3
b
3
)
1
In this way, it makes sense to express both x
2
and x
3
as functions of x
1
:
x
2
b
2
=

p
1
p
2
(x
1
b
1
)
x
3
b
3
=
1

p
1
p
3
(x
1
b
1
)
Now, we can replace in the fourth FOC,
ln (x
1
b
1
) + ln (x
2
b
2
) + (1 ) ln (x
3
b
3
) = u
ln (x
1
b
1
) + ln
_

p
1
p
2
(x
1
b
1
)
_
+ (1 ) ln
_
1

p
1
p
3
(x
1
b
1
)
_
= u
ln (x
1
b
1
) + ln
_

p
1
p
2
_
+ (1 ) ln
_
1

p
1
p
3
_
= u
Then,
h
1
(p, u) = b
1
+
_
_
p
1

_
(+)
_
p
2

_
p
3

_
e
u
In the same way,
h
2
(p, u) = b
2
+
_
_
p
2

_
(+) _
p
1

_
p
3

_
e
u
5
h
3
(p, u) = b
3
+
_
_
p
3

_
(+) _
p
1

_
p
2

_
e
u
Then, the expenditure function is given by:
e(p, u) = p
1
h
1
(p, u) + p
2
h
2
(p, u) + p
3
h
3
(p, u)
e(p, u) = p
1
_
b
1
+
_
_
p
1

_
(+)
_
p
2

_
p
3

_
e
u
_
+ p
2
_
b
2
+
_
_
p
2

_
(+) _
p
1

_
p
3

_
e
u
_
+ p
3
_
b
3
+
_
_
p
3

_
(+) _
p
1

_
p
2

_
e
u
_
= p
1
b
1
+ p
2
b
2
+ p
3
b
3
+
_
p

1
()
+
_
p
2

_
p
3

_
e
u
+
_
p

2
()
+
_
p
1

_
p
3

_
e
u
+
_
p

3
()
+
_
p
1

_
p
2

_
e
u
= p
1
b
1
+ p
2
b
2
+ p
3
b
3
+ e
u
p

1
p

2
p

3

_
+
_

_
+
_

+
_

__
= p
1
b
1
+ p
2
b
2
+ p
3
b
3
+ e
u
p

1
p

2
p

3

_
+
_

_
+
_

_
+
_

__
= p
1
b
1
+ p
2
b
2
+ p
3
b
3
+ e
u
p

1
p

2
p

3
_

_
+
_

+
_
+
_

+
_
+
_

+
__
= p
1
b
1
+ p
2
b
2
+ p
3
b
3
+ e
u
p

1
p

2
p

3
_

_
( + + )
So, after all this long algebra, we say:
e(p, u) = p
1
b
1
+ p
2
b
2
+ p
3
b
3
+ e
u
_

_
p

1
p

2
p

3
Next, we check that the properties from Proposition 3.E.2 hold:
i. Homogeneity of degree one in p:
e(tp, u) = tp
1
b
1
+ tp
2
b
2
+ tp
3
b
3
+ e
u
_

_
(tp
1
)

(tp
2
)

(tp
3
)

= t (p
1
b
1
+ p
2
b
2
+ p
3
b
3
) + e
u
_

_
p

1
p

2
p

3
t
++
= t
_
p
1
b
1
+ p
2
b
2
+ p
3
b
3
+ e
u
_

_
p

1
p

2
p

3
_
= te(p, u)
6
ii. Strictly increasing in u and non decreasing in p
l
for any l. It is straightforward for
u. Let us do it for p
1
(it would be similar for p
2
and p
3
):
e(p, u)
p
1
= b
1
+ p
1
1
e
u
_

_
p

1
p

2
p

3
> 0
Note we re implicitly assuming b
1
> 0.
iii. Concavity in p: A linear function is concave, and also p

1
p

2
p

3
is concave. Then,
e(p, u) is concave.
iv. Continuity. Each part is continuous. So e(p, u) is continuous.
Finally, we check that the properties from Proposition 3.E.3 hold:
i. Homogeneity of degree zero in p. We do it for h
1
(p, u). The other ones are similar.
h
1
(tp, u) = b
1
+
_
_
tp
1

_
(+)
_
tp
2

_
tp
3

_
e
u
= b
1
+ t
(+)
t

_
_
p
1

_
(+)
_
p
2

_
p
3

_
e
u
= b
1
+
_
_
p
1

_
(+)
_
p
2

_
p
3

_
e
u
= h
1
(p, u)
ii. No excess utility:
u(h(p, u)) = u (h
1
(p, u), h
2
(p, u), h
3
(p, u))
= ln (h
1
(p, u) b
1
) + ln (h
2
(p, u) b
2
) + ln (h
3
(p, u) b
3
)
= ln
_
b
1
+
_
_
p
1

_
(+)
_
p
2

_
p
3

_
e
u
b
1
_
+ ln
_
b
2
+
_
_
p
2

_
(+) _
p
1

_
p
3

_
e
u
b
2
_
+ ln
_
b
3
+
_
_
p
3

_
(+) _
p
1

_
p
2

_
e
u
b
3
_
= ln
__
_
p
1

_
(+)
_
p
2

_
p
3

_
e
u
_
+ ln
__
_
p
2

_
(+) _
p
1

_
p
3

_
e
u
_
+ ln
__
_
p
3

_
(+) _
p
1

_
p
2

_
e
u
_
= ln
_
_
p
1

_
(+)
_
p
2

_
p
3

_
+ u
+ ln
_
_
p
2

_
(+) _
p
1

_
p
3

_
+ u
7
+ ln
_
_
p
3

_
(+) _
p
1

_
p
2

_
+ u
= u + ln
_ _
p
1

_
(+)
_
p
2

_
p
3

_
p
2

_
(+) _
p
1

_
p
3

_
p
3

_
(+) _
p
1

_
p
2

_
_
= u
iii. Uniqueness. We found the function. It gives a unique value.
(b) Show that the derivative of the expenditure function are the Hicksean demand func-
tions you derived in (a).
Answer: We know now that
e(p, u) = p
1
b
1
+ p
2
b
2
+ p
3
b
3
+ e
u
_

_
p

1
p

2
p

3
We do it for h
1
(p, u). The other ones are similar.
e(p, u)
p
1
= b
1


p
1
e
u
_

_
p

1
p

2
p

3
= h
1
(p, u)
(c) Verify that the Slutsky equation holds.
Answer: I will do a own price derivative and a cross price derivative. Take
h
1
(p, u) = b
1
+
_
_
p
1

_
(+)
_
p
2

_
p
3

_
e
u
x
1
(p, w) = (1 ) b
1
+ p
1
1
(w p
2
b
2
p
3
b
3
)
Then,
x
1
(p, w)
p
1
=
h
1
(p, w)
p
1

x
1
(p, w)
w
x
1
(p, w) for u = v(p, w)
p
1
1
p
1
1
(w p
2
b
2
p
3
b
3
) = ( + )p
1
1
_
_
p
1

_
(+)
_
p
2

_
p
3

_
e
u
p
1
1
x
1
p
1
1
(w p
2
b
2
p
3
b
3
) = ( + )
_
_
p
1

_
(+)
_
p
2

_
p
3

_
e
u
+ x
1
(1 )b
1
+ p
1
1
(w p
2
b
2
p
3
b
3
) = (1 )b
1
+ (1 )
_
_
p
1

_
(+)
_
p
2

_
p
3

_
e
u
+ x
1
(1 )b
1
+ p
1
1
(w p
2
b
2
p
3
b
3
) = (1 )
_
b
1
+
_
p
1

_
(+)
_
p
2

_
p
3

e
u
_
+ x
1
x
1
(p, w) = (1 )h
1
(p, u) + x
1
(p, w) for u = v(p, w)
x
1
(p, w) = (1 )h
1
(p, v(p, w)) + x
1
(p, w)
8
x
1
(p, w) = (1 )x
1
(p, w) + x
1
(p, w)
x
1
(p, w) = x
1
(p, w)
We are done then.
Lets do now a cross price derivative.
x
1
(p, w)
p
2
=
h
1
(p, w)
p
2

x
1
(p, w)
w
x
2
(p, w) for u = v(p, w)
p
1
1
b
2
= p
1
2
_
_
p
1

_
(+)
_
p
2

_
p
3

_
e
u
p
1
1
x
2
p
1
1
b
2
= p
1
2
_
_
p
1

_
(+)
_
p
2

_
p
3

_
e
u
+ p
1
1
x
2
b
2
= p
1
p
1
2
_
p
1

_
_
p
1

_
p
1

_
(+)
_
p
2

_
p
3

_
e
u
+ x
2
b
2
= p
1
p
1
2
_
p
1

_
1
_
_
p
1

_
p
2

_
p
3

_
e
u
+ x
2
b
2
= p
1
2
_
_
p
1

_
p
2

_
p
3

_
e
u
+ x
2
b
2
=
_
p
2

_
1
_
_
p
1

_
p
2

_
p
3

_
e
u
+ x
2
b
2
=
_
_
p
1

_
p
2

_
(1)
_
p
3

_
e
u
+ x
2
b
2
=
_
_
p
1

_
p
2

_
(+)
_
p
3

_
e
u
+ x
2
Now, recall that:
h
2
(p, u) = b
2
+
_
_
p
2

_
(+) _
p
1

_
p
3

_
e
u
Then,
b
2
= (h
2
(p, u) b
2
) + x
2
(p, w) for u = v(p, w)
h
2
(p, u) = x
2
(p, w) for u = v(p, w)
h
2
(p, v(p, w)) = x
2
(p, w)
We are done.
(d) Verify that the own-substitution terms are negative and that compensated cross-price
effects are symmetric.
Answer:
Take again:
h
1
(p, u) = b
1
+
_
_
p
1

_
(+)
_
p
2

_
p
3

_
e
u
9
h
2
(p, u) = b
2
+
_
_
p
2

_
(+) _
p
1

_
p
3

_
e
u
We have shown already that
h
1
p
1
< 1. So we are left to show that:
h
1
p
2
=
h
2
p
1
p
1
2
_
_
p
1

_
(+)
_
p
2

_
p
3

_
e
u
= p
1
1
_
_
p
2

_
(+) _
p
1

_
p
3

_
e
u
_
p
2

_
1 _
p
1

_
(+)
_
p
2

_
p
3

=
_
p
1

_
1
_
p
2

_
(+) _
p
1

_
p
3

_
p
1

_
(+)
_
p
2
1
_

_
p
3

=
_
p
1

_
1
_
p
2

_
(+)
_
p
3

We are done.
10
3. MWG 3.E.5
B
Show that if u(x) is homogeneous of degree one, then h(p, u) and e(p, u) are
homogeneous of degree one in u [i.e., they can be written as h(p, u) =

h(p)u and e(p, u) =
e(p)u].
Answer: We know that the utility function is homogeneous of degree one, so u(x) = u(x).
Let h(p, u) be the hicksean demand at prices p and utility level u. This means that:
ph(p, u) px for all x such that u(x) u
Now let prices be p again but utility level be u. If the hicksean demand is homogeneous
of degree one, h(p, u) = h(p, u). We will show that is true by contradiction. Assume it is
not. Then, there exists x with p x < ph(p, u) and u ( x) u. Since utility is homogeneous,
u
_
x

_
u. To sum up,
p
x

< ph(p, u) and u


_
x

_
u
which means that h(p, u) does not minimize expenditure. We have arrived to a contradiction
and thus, we have shown that:
h(p, u) = h(p, u)
In particular let = u
1
and dene

h(p) h(p, 1). Then,
h(p, u) =
1

h (p, u) = uh
_
p,
1
u
u
_
= h (p, 1) u =

h(p)u
By denition,
e(p, u) = ph(p, u) = p
_

h(p)u
_
=
_
p

h(p)
_
u = e(p)u
by letting e(p) p

h(p).
11
4. MWG 3.F.2
A
Show by means of a graphic example that the separating hyperplane theorem
does not hold for nonconvex sets. Then argue that if K is closed but not convex, there is
always some x K that cannot be separated from K.
Answer: Figure 1 depicts a nonconvex set, and a point that cannot be separated with a
hyperplane.
K
x K
Figure 1: Mrs. Pacman
Let K be closed and nonconvex. This means there exists x, x

K and (0, 1) such that


x

= x + (1 )x

and x

K. Assume that p R
L
and c R exist such that x

can be
separated from K. Then,
px

< c px for all x K


Then,
p
_
x + (1 )x

< c
but
px c and px

c
Then,
px c and p(1 )x

(1 )c
which implies:
p
_
x + (1 )x

c
We have reached a contradiction.
12
5. MWG3.G.4
B
A utility function u(x) is additively separable if it has the form u(x) =
l
u
l
(x
l
).
(a) Show that additive separability is a cardinal property that is preserved only under
linear transformations of the utility function.
Answer: First, let v(x) = a + bu(x) be a linear transformation of the utility function,
with a R and b > 0. Then
v(x) = a +

l
bu
l
(x
l
) =

l
v
l
(x
l
)
where v
1
(x
1
) = a + bu
1
(x
1
) and v
l
(x
l
) = bu
l
(x
l
) for all l = 1. Then, any linear
transformation of an additively separable utility function also generates an additively
separable utility function. Note, also that a transformation v(x) = (u(x))
2
does not
generate an additively separable utility function.
(b) Show that the induced ordering on any group of commodities is independent of what-
ever xed values we attach to the remaining ones. It turns out that this ordinal property
is not only necessary but also sufcient for the existence of an additive separable repre-
sentation. [You should not attempt a proof. This is very hard. See Debreu (1960)]
Answer: This is similar to what we have done in the midterm. Take a subgroup of
commodities

L. We can write the utility function as follows:
u
_
x

L
, x

L
_
=

L
u
l
(x
l
) +

L
u
l
(x
l
)
where x

L
is a vector of commodities in group

L and x

L
is the vector of the remain-
ing commodities. Take
_
x

L
, x

L
_
and
_
x

L
, x

L
_
, so the only difference between these
bundles is on the set of commodities

L. Then,
u
_
x

L
, x

L
_
u
_
x

L
, x

L
_
=

L
u
l
(x
l
) +

L
u
l
(x
l
)
_
_

L
u
l
_
x

l
_
+

L
u
l
(x
l
)
_
_
=

L
u
l
(x
l
)

L
u
l
_
x

l
_
In this way, we can see that the induced ordering on any group of commodities is
independent of whatever xed values we attach to the remaining ones.
(c) Showthat the Walrasian and Hicksean demand functions generated by additively sepa-
rable utility functions admit no inferior goods if the functions u
l
() are strictly concave.
(You can assume differentiability and interiority to answer this question.)
Answer: In an interior equilibrium, the following condition holds with equality:
u
l
(x
l
)
x
l
= p
l
First, note that when wealth increases, must fall. In other words, note that
(p, w)
w
< 0
13
Assume it is not. Then, for the above condition to still hold, the consumer has to con-
sume less of all goods. In such a case, Walras Law wouldnt hold. Let us rewrite the
FOC as an identity, and derive with respect to w:
u
l
(x
l
(p, w))
x
l
(p, w)p
l

2
u
l
(x
l
(p, w))
x
2
l
x
l
(p, w)
w

(p, w)
w
p
l
Then,
x
l
(p, w)
w
> 0 for all l
(d) (Harder) Suppose that all u
l
() are identical and twice differentiable. Let u() = u
l
().
Show that if
_
t u

(t)
u

(t)
_
< 1 for all t, then the Walrasian demand x(p, w) has the so-called
gross substitute property, i.e.,
x
l
(p,w)
p
k
> 0 for all l and k = l.
Answer: This is longer and more boring:

l
p
l
x
l
(p, w) = w

l
_
p
l
x
l
(p, w)
p
k
_
+ x
k
(p, w) = 0
But,
u(x
l
(p, w))
x
l
(p, w)p
l

2
u(x
l
(p, w))
x
2
l
x
l
(p, w)
p
k

(p, w)
p
k
p
l
x
l
(p, w)
p
k

(p,w)
p
k
p
l

2
u(x
l
(p,w))
x
2
l
And,

2
u(x
l
(p, w))
x
2
l
x
l
(p, w)
p
l

(p, w)
p
k
p
l
+ (p, w)
x
l
(p, w)
p
l

(p,w)
p
k
p
l
+ (p, w)

2
u(x
l
(p,w))
x
2
l
Then,

l
_
p
l
x
l
(p, w)
p
k
_
+ x
k
(p, w) = 0

l
_
_
p
l
(p,w)
p
k
p
l

2
u(x
l
(p,w))
x
2
l
_
_
+ p
k
(p, w)

2
u(x
k
(p,w))
x
2
k
+ x
k
(p, w) = 0
14

l
_
_
p
l
(p,w)
p
k
p
l

2
u(x
l
(p,w))
x
2
l
_
_
+
u(x
k
(p,w))
x
k

2
u(x
k
(p,w))
x
2
k
+ x
k
(p, w) = 0
(p, w)
p
k

l
_
_
p
2
l

2
u(x
l
(p,w))
x
2
l
_
_
+
u(x
k
(p,w))
x
k

2
u(x
k
(p,w))
x
2
k
+ x
k
(p, w) = 0
(p, w)
p
k

l
_
_
p
2
l

2
u(x
l
(p,w))
x
2
l
_
_
+
_
_
u(x
k
(p,w))
x
k
x
k
(p, w)

2
u(x
k
(p,w))
x
2
k
+ 1
_
_
x
k
(p, w) = 0
By the assumption in this exercise,
_
_
u(x
k
(p,w))
x
k
x
k
(p, w)

2
u(x
k
(p,w))
x
2
k
+ 1
_
_
> 0
Moreover, since the function is strictly concave,

l
_
_
p
2
l

2
u(x
l
(p,w))
x
2
l
_
_
< 0
As a result,
(p, w)
p
k
< 0
But again, since
x
l
(p, w)
p
k

(p,w)
p
k
p
l

2
u(x
l
(p,w))
x
2
l
Then,
x
l
(p, w)
p
k
> 0
which is what we wanted to show.
15

También podría gustarte